Find $lim_{nto infty} int_{mathbb R}frac{n log^{4}(x)}{n+nx+x^2}chi_{[0,infty[}dlambda(x)$












1












$begingroup$


$lim_{nto infty} int_{mathbb R}frac{n log^{4}(x)}{n+nx+x^2}chi_{[0,infty[}dlambda(x)$



This seems like an apt situation to utilize dominating convergence.



$f_{n}(x):=frac{n log^{4}(x)}{n+nx+x^2}$ is continuous on $]0,infty[$ and therefore measurable $forall n in mathbb N$. But I am struggling to show $int_{[0,infty[}frac{n log^{4}(x)}{n+nx+x^2}dlambda(x)=int_{]0,infty[}frac{n log^{4}(x)}{n+nx+x^2}dlambda(x)<infty$ to ensure $(f_{n})_{n}subseteqmathcal{L}^{1}(mu)$.



$|frac{n log^{4}(x)}{n+nx+x^2}|leq...?$



Any hints?










share|cite|improve this question









$endgroup$








  • 1




    $begingroup$
    Hint: The integrand monotone-increases to $(log x)^4/(1+x) mathbf{1}_{[0,infty)}(x)$ as $ntoinfty$.
    $endgroup$
    – Sangchul Lee
    Dec 4 '18 at 20:27












  • $begingroup$
    Is there a reason why you repeatedly use, for various functions $g$, the incorrect formula $$int_{mathbb R}g(x)chi_{[0,infty[}dlambda(x)$$ rather than the correct $$int_0^infty g(x)dx ?$$
    $endgroup$
    – Did
    Dec 4 '18 at 20:46






  • 1




    $begingroup$
    I was of the view $int_{mathbb R}g(x)chi_{[0,infty[}dlambda(x)=int_{[0,infty[}g(x)dlambda(x)$ as the lebesgue integral rather than $int^{infty}_{0}g(x)dx$ which we defined as the Riemann integral
    $endgroup$
    – SABOY
    Dec 4 '18 at 20:49








  • 1




    $begingroup$
    maybe you mean that you are searching an estimate for the integral $int_{[0,infty)}frac{(log x)^4}{1+x},lambda(dx)$? You have one in the answer of @p4sch, just fill the details, find a rough estimation of the integral in the set $[0,e]$, just note that the integrand is positive and that $lim_{xto 0^+}frac{(log x)^4}{1+x}=infty$, at least the integral is positive here
    $endgroup$
    – Masacroso
    Dec 4 '18 at 23:01






  • 1




    $begingroup$
    @SABOY I dont know your version of Fatou's lemma, I just know the version of above. Probably your version of Fatou's lemma is extended to a measure space $(X,mu,overline{Bbb R})$ instead of $(X,mu,overline{Bbb R}^+)$. In this case the condition $gle f_n$ seems to be added to ensure that, by the dominated convergence theorem, the integral $lim_nint_X f_n^-,dmu=int_Xlim f_n^-, dmu$, where $f_n^-$ is the negative part of $f_n$. Anyway Im just guessing, Idk really the reasons on your version of Fatou's lemma. In you case you can choose $g=0$
    $endgroup$
    – Masacroso
    Dec 4 '18 at 23:07


















1












$begingroup$


$lim_{nto infty} int_{mathbb R}frac{n log^{4}(x)}{n+nx+x^2}chi_{[0,infty[}dlambda(x)$



This seems like an apt situation to utilize dominating convergence.



$f_{n}(x):=frac{n log^{4}(x)}{n+nx+x^2}$ is continuous on $]0,infty[$ and therefore measurable $forall n in mathbb N$. But I am struggling to show $int_{[0,infty[}frac{n log^{4}(x)}{n+nx+x^2}dlambda(x)=int_{]0,infty[}frac{n log^{4}(x)}{n+nx+x^2}dlambda(x)<infty$ to ensure $(f_{n})_{n}subseteqmathcal{L}^{1}(mu)$.



$|frac{n log^{4}(x)}{n+nx+x^2}|leq...?$



Any hints?










share|cite|improve this question









$endgroup$








  • 1




    $begingroup$
    Hint: The integrand monotone-increases to $(log x)^4/(1+x) mathbf{1}_{[0,infty)}(x)$ as $ntoinfty$.
    $endgroup$
    – Sangchul Lee
    Dec 4 '18 at 20:27












  • $begingroup$
    Is there a reason why you repeatedly use, for various functions $g$, the incorrect formula $$int_{mathbb R}g(x)chi_{[0,infty[}dlambda(x)$$ rather than the correct $$int_0^infty g(x)dx ?$$
    $endgroup$
    – Did
    Dec 4 '18 at 20:46






  • 1




    $begingroup$
    I was of the view $int_{mathbb R}g(x)chi_{[0,infty[}dlambda(x)=int_{[0,infty[}g(x)dlambda(x)$ as the lebesgue integral rather than $int^{infty}_{0}g(x)dx$ which we defined as the Riemann integral
    $endgroup$
    – SABOY
    Dec 4 '18 at 20:49








  • 1




    $begingroup$
    maybe you mean that you are searching an estimate for the integral $int_{[0,infty)}frac{(log x)^4}{1+x},lambda(dx)$? You have one in the answer of @p4sch, just fill the details, find a rough estimation of the integral in the set $[0,e]$, just note that the integrand is positive and that $lim_{xto 0^+}frac{(log x)^4}{1+x}=infty$, at least the integral is positive here
    $endgroup$
    – Masacroso
    Dec 4 '18 at 23:01






  • 1




    $begingroup$
    @SABOY I dont know your version of Fatou's lemma, I just know the version of above. Probably your version of Fatou's lemma is extended to a measure space $(X,mu,overline{Bbb R})$ instead of $(X,mu,overline{Bbb R}^+)$. In this case the condition $gle f_n$ seems to be added to ensure that, by the dominated convergence theorem, the integral $lim_nint_X f_n^-,dmu=int_Xlim f_n^-, dmu$, where $f_n^-$ is the negative part of $f_n$. Anyway Im just guessing, Idk really the reasons on your version of Fatou's lemma. In you case you can choose $g=0$
    $endgroup$
    – Masacroso
    Dec 4 '18 at 23:07
















1












1








1





$begingroup$


$lim_{nto infty} int_{mathbb R}frac{n log^{4}(x)}{n+nx+x^2}chi_{[0,infty[}dlambda(x)$



This seems like an apt situation to utilize dominating convergence.



$f_{n}(x):=frac{n log^{4}(x)}{n+nx+x^2}$ is continuous on $]0,infty[$ and therefore measurable $forall n in mathbb N$. But I am struggling to show $int_{[0,infty[}frac{n log^{4}(x)}{n+nx+x^2}dlambda(x)=int_{]0,infty[}frac{n log^{4}(x)}{n+nx+x^2}dlambda(x)<infty$ to ensure $(f_{n})_{n}subseteqmathcal{L}^{1}(mu)$.



$|frac{n log^{4}(x)}{n+nx+x^2}|leq...?$



Any hints?










share|cite|improve this question









$endgroup$




$lim_{nto infty} int_{mathbb R}frac{n log^{4}(x)}{n+nx+x^2}chi_{[0,infty[}dlambda(x)$



This seems like an apt situation to utilize dominating convergence.



$f_{n}(x):=frac{n log^{4}(x)}{n+nx+x^2}$ is continuous on $]0,infty[$ and therefore measurable $forall n in mathbb N$. But I am struggling to show $int_{[0,infty[}frac{n log^{4}(x)}{n+nx+x^2}dlambda(x)=int_{]0,infty[}frac{n log^{4}(x)}{n+nx+x^2}dlambda(x)<infty$ to ensure $(f_{n})_{n}subseteqmathcal{L}^{1}(mu)$.



$|frac{n log^{4}(x)}{n+nx+x^2}|leq...?$



Any hints?







real-analysis integration measure-theory convergence






share|cite|improve this question













share|cite|improve this question











share|cite|improve this question




share|cite|improve this question










asked Dec 4 '18 at 20:20









SABOYSABOY

588311




588311








  • 1




    $begingroup$
    Hint: The integrand monotone-increases to $(log x)^4/(1+x) mathbf{1}_{[0,infty)}(x)$ as $ntoinfty$.
    $endgroup$
    – Sangchul Lee
    Dec 4 '18 at 20:27












  • $begingroup$
    Is there a reason why you repeatedly use, for various functions $g$, the incorrect formula $$int_{mathbb R}g(x)chi_{[0,infty[}dlambda(x)$$ rather than the correct $$int_0^infty g(x)dx ?$$
    $endgroup$
    – Did
    Dec 4 '18 at 20:46






  • 1




    $begingroup$
    I was of the view $int_{mathbb R}g(x)chi_{[0,infty[}dlambda(x)=int_{[0,infty[}g(x)dlambda(x)$ as the lebesgue integral rather than $int^{infty}_{0}g(x)dx$ which we defined as the Riemann integral
    $endgroup$
    – SABOY
    Dec 4 '18 at 20:49








  • 1




    $begingroup$
    maybe you mean that you are searching an estimate for the integral $int_{[0,infty)}frac{(log x)^4}{1+x},lambda(dx)$? You have one in the answer of @p4sch, just fill the details, find a rough estimation of the integral in the set $[0,e]$, just note that the integrand is positive and that $lim_{xto 0^+}frac{(log x)^4}{1+x}=infty$, at least the integral is positive here
    $endgroup$
    – Masacroso
    Dec 4 '18 at 23:01






  • 1




    $begingroup$
    @SABOY I dont know your version of Fatou's lemma, I just know the version of above. Probably your version of Fatou's lemma is extended to a measure space $(X,mu,overline{Bbb R})$ instead of $(X,mu,overline{Bbb R}^+)$. In this case the condition $gle f_n$ seems to be added to ensure that, by the dominated convergence theorem, the integral $lim_nint_X f_n^-,dmu=int_Xlim f_n^-, dmu$, where $f_n^-$ is the negative part of $f_n$. Anyway Im just guessing, Idk really the reasons on your version of Fatou's lemma. In you case you can choose $g=0$
    $endgroup$
    – Masacroso
    Dec 4 '18 at 23:07
















  • 1




    $begingroup$
    Hint: The integrand monotone-increases to $(log x)^4/(1+x) mathbf{1}_{[0,infty)}(x)$ as $ntoinfty$.
    $endgroup$
    – Sangchul Lee
    Dec 4 '18 at 20:27












  • $begingroup$
    Is there a reason why you repeatedly use, for various functions $g$, the incorrect formula $$int_{mathbb R}g(x)chi_{[0,infty[}dlambda(x)$$ rather than the correct $$int_0^infty g(x)dx ?$$
    $endgroup$
    – Did
    Dec 4 '18 at 20:46






  • 1




    $begingroup$
    I was of the view $int_{mathbb R}g(x)chi_{[0,infty[}dlambda(x)=int_{[0,infty[}g(x)dlambda(x)$ as the lebesgue integral rather than $int^{infty}_{0}g(x)dx$ which we defined as the Riemann integral
    $endgroup$
    – SABOY
    Dec 4 '18 at 20:49








  • 1




    $begingroup$
    maybe you mean that you are searching an estimate for the integral $int_{[0,infty)}frac{(log x)^4}{1+x},lambda(dx)$? You have one in the answer of @p4sch, just fill the details, find a rough estimation of the integral in the set $[0,e]$, just note that the integrand is positive and that $lim_{xto 0^+}frac{(log x)^4}{1+x}=infty$, at least the integral is positive here
    $endgroup$
    – Masacroso
    Dec 4 '18 at 23:01






  • 1




    $begingroup$
    @SABOY I dont know your version of Fatou's lemma, I just know the version of above. Probably your version of Fatou's lemma is extended to a measure space $(X,mu,overline{Bbb R})$ instead of $(X,mu,overline{Bbb R}^+)$. In this case the condition $gle f_n$ seems to be added to ensure that, by the dominated convergence theorem, the integral $lim_nint_X f_n^-,dmu=int_Xlim f_n^-, dmu$, where $f_n^-$ is the negative part of $f_n$. Anyway Im just guessing, Idk really the reasons on your version of Fatou's lemma. In you case you can choose $g=0$
    $endgroup$
    – Masacroso
    Dec 4 '18 at 23:07










1




1




$begingroup$
Hint: The integrand monotone-increases to $(log x)^4/(1+x) mathbf{1}_{[0,infty)}(x)$ as $ntoinfty$.
$endgroup$
– Sangchul Lee
Dec 4 '18 at 20:27






$begingroup$
Hint: The integrand monotone-increases to $(log x)^4/(1+x) mathbf{1}_{[0,infty)}(x)$ as $ntoinfty$.
$endgroup$
– Sangchul Lee
Dec 4 '18 at 20:27














$begingroup$
Is there a reason why you repeatedly use, for various functions $g$, the incorrect formula $$int_{mathbb R}g(x)chi_{[0,infty[}dlambda(x)$$ rather than the correct $$int_0^infty g(x)dx ?$$
$endgroup$
– Did
Dec 4 '18 at 20:46




$begingroup$
Is there a reason why you repeatedly use, for various functions $g$, the incorrect formula $$int_{mathbb R}g(x)chi_{[0,infty[}dlambda(x)$$ rather than the correct $$int_0^infty g(x)dx ?$$
$endgroup$
– Did
Dec 4 '18 at 20:46




1




1




$begingroup$
I was of the view $int_{mathbb R}g(x)chi_{[0,infty[}dlambda(x)=int_{[0,infty[}g(x)dlambda(x)$ as the lebesgue integral rather than $int^{infty}_{0}g(x)dx$ which we defined as the Riemann integral
$endgroup$
– SABOY
Dec 4 '18 at 20:49






$begingroup$
I was of the view $int_{mathbb R}g(x)chi_{[0,infty[}dlambda(x)=int_{[0,infty[}g(x)dlambda(x)$ as the lebesgue integral rather than $int^{infty}_{0}g(x)dx$ which we defined as the Riemann integral
$endgroup$
– SABOY
Dec 4 '18 at 20:49






1




1




$begingroup$
maybe you mean that you are searching an estimate for the integral $int_{[0,infty)}frac{(log x)^4}{1+x},lambda(dx)$? You have one in the answer of @p4sch, just fill the details, find a rough estimation of the integral in the set $[0,e]$, just note that the integrand is positive and that $lim_{xto 0^+}frac{(log x)^4}{1+x}=infty$, at least the integral is positive here
$endgroup$
– Masacroso
Dec 4 '18 at 23:01




$begingroup$
maybe you mean that you are searching an estimate for the integral $int_{[0,infty)}frac{(log x)^4}{1+x},lambda(dx)$? You have one in the answer of @p4sch, just fill the details, find a rough estimation of the integral in the set $[0,e]$, just note that the integrand is positive and that $lim_{xto 0^+}frac{(log x)^4}{1+x}=infty$, at least the integral is positive here
$endgroup$
– Masacroso
Dec 4 '18 at 23:01




1




1




$begingroup$
@SABOY I dont know your version of Fatou's lemma, I just know the version of above. Probably your version of Fatou's lemma is extended to a measure space $(X,mu,overline{Bbb R})$ instead of $(X,mu,overline{Bbb R}^+)$. In this case the condition $gle f_n$ seems to be added to ensure that, by the dominated convergence theorem, the integral $lim_nint_X f_n^-,dmu=int_Xlim f_n^-, dmu$, where $f_n^-$ is the negative part of $f_n$. Anyway Im just guessing, Idk really the reasons on your version of Fatou's lemma. In you case you can choose $g=0$
$endgroup$
– Masacroso
Dec 4 '18 at 23:07






$begingroup$
@SABOY I dont know your version of Fatou's lemma, I just know the version of above. Probably your version of Fatou's lemma is extended to a measure space $(X,mu,overline{Bbb R})$ instead of $(X,mu,overline{Bbb R}^+)$. In this case the condition $gle f_n$ seems to be added to ensure that, by the dominated convergence theorem, the integral $lim_nint_X f_n^-,dmu=int_Xlim f_n^-, dmu$, where $f_n^-$ is the negative part of $f_n$. Anyway Im just guessing, Idk really the reasons on your version of Fatou's lemma. In you case you can choose $g=0$
$endgroup$
– Masacroso
Dec 4 '18 at 23:07












2 Answers
2






active

oldest

votes


















2












$begingroup$

Note that for $x > 0$ we have
$$f_n(x):= frac{log(x)^4}{1+x+x^2/n}.$$
Thus for all $x >0$
$$f_n(x) le f_{n+1}(x) le f(x):=frac{log(x)^4}{1+x}$$
and pointwise $f_n rightarrow f$. Therefore, we can apply the monotone convergence theorem. However, the limes $f$ is not integrable, because
$$int_{e}^infty f(x) , dx ge int_{e}^infty frac{1}{1+x} , dx =infty,$$
and we cannot apply the dominated convergence theorem.






share|cite|improve this answer









$endgroup$





















    1












    $begingroup$

    To apply LDCT you need an $L^1$ majorant $g$ satisfying $|f_n(x)| le g(x)$ almost everywhere for all $n$. If $f_n to f$ almost everywhere then you have $|f(x)| le g(x)$ too.



    You can write $$f_n(x) = frac{n log^4 x}{n + nx + x^2} = frac{log^4 x}{1 + x + x^2/n}$$ so that $$f_n(x) to frac{log^4 x}{1+x}.$$
    This function is not integrable, so you won't find an integrable majorant $g$ with $|f_n| le g$ for all $n$. It looks like LDCT may not be the way to go.






    share|cite|improve this answer









    $endgroup$













      Your Answer





      StackExchange.ifUsing("editor", function () {
      return StackExchange.using("mathjaxEditing", function () {
      StackExchange.MarkdownEditor.creationCallbacks.add(function (editor, postfix) {
      StackExchange.mathjaxEditing.prepareWmdForMathJax(editor, postfix, [["$", "$"], ["\\(","\\)"]]);
      });
      });
      }, "mathjax-editing");

      StackExchange.ready(function() {
      var channelOptions = {
      tags: "".split(" "),
      id: "69"
      };
      initTagRenderer("".split(" "), "".split(" "), channelOptions);

      StackExchange.using("externalEditor", function() {
      // Have to fire editor after snippets, if snippets enabled
      if (StackExchange.settings.snippets.snippetsEnabled) {
      StackExchange.using("snippets", function() {
      createEditor();
      });
      }
      else {
      createEditor();
      }
      });

      function createEditor() {
      StackExchange.prepareEditor({
      heartbeatType: 'answer',
      autoActivateHeartbeat: false,
      convertImagesToLinks: true,
      noModals: true,
      showLowRepImageUploadWarning: true,
      reputationToPostImages: 10,
      bindNavPrevention: true,
      postfix: "",
      imageUploader: {
      brandingHtml: "Powered by u003ca class="icon-imgur-white" href="https://imgur.com/"u003eu003c/au003e",
      contentPolicyHtml: "User contributions licensed under u003ca href="https://creativecommons.org/licenses/by-sa/3.0/"u003ecc by-sa 3.0 with attribution requiredu003c/au003e u003ca href="https://stackoverflow.com/legal/content-policy"u003e(content policy)u003c/au003e",
      allowUrls: true
      },
      noCode: true, onDemand: true,
      discardSelector: ".discard-answer"
      ,immediatelyShowMarkdownHelp:true
      });


      }
      });














      draft saved

      draft discarded


















      StackExchange.ready(
      function () {
      StackExchange.openid.initPostLogin('.new-post-login', 'https%3a%2f%2fmath.stackexchange.com%2fquestions%2f3026102%2ffind-lim-n-to-infty-int-mathbb-r-fracn-log4xnnxx2-chi-0%23new-answer', 'question_page');
      }
      );

      Post as a guest















      Required, but never shown

























      2 Answers
      2






      active

      oldest

      votes








      2 Answers
      2






      active

      oldest

      votes









      active

      oldest

      votes






      active

      oldest

      votes









      2












      $begingroup$

      Note that for $x > 0$ we have
      $$f_n(x):= frac{log(x)^4}{1+x+x^2/n}.$$
      Thus for all $x >0$
      $$f_n(x) le f_{n+1}(x) le f(x):=frac{log(x)^4}{1+x}$$
      and pointwise $f_n rightarrow f$. Therefore, we can apply the monotone convergence theorem. However, the limes $f$ is not integrable, because
      $$int_{e}^infty f(x) , dx ge int_{e}^infty frac{1}{1+x} , dx =infty,$$
      and we cannot apply the dominated convergence theorem.






      share|cite|improve this answer









      $endgroup$


















        2












        $begingroup$

        Note that for $x > 0$ we have
        $$f_n(x):= frac{log(x)^4}{1+x+x^2/n}.$$
        Thus for all $x >0$
        $$f_n(x) le f_{n+1}(x) le f(x):=frac{log(x)^4}{1+x}$$
        and pointwise $f_n rightarrow f$. Therefore, we can apply the monotone convergence theorem. However, the limes $f$ is not integrable, because
        $$int_{e}^infty f(x) , dx ge int_{e}^infty frac{1}{1+x} , dx =infty,$$
        and we cannot apply the dominated convergence theorem.






        share|cite|improve this answer









        $endgroup$
















          2












          2








          2





          $begingroup$

          Note that for $x > 0$ we have
          $$f_n(x):= frac{log(x)^4}{1+x+x^2/n}.$$
          Thus for all $x >0$
          $$f_n(x) le f_{n+1}(x) le f(x):=frac{log(x)^4}{1+x}$$
          and pointwise $f_n rightarrow f$. Therefore, we can apply the monotone convergence theorem. However, the limes $f$ is not integrable, because
          $$int_{e}^infty f(x) , dx ge int_{e}^infty frac{1}{1+x} , dx =infty,$$
          and we cannot apply the dominated convergence theorem.






          share|cite|improve this answer









          $endgroup$



          Note that for $x > 0$ we have
          $$f_n(x):= frac{log(x)^4}{1+x+x^2/n}.$$
          Thus for all $x >0$
          $$f_n(x) le f_{n+1}(x) le f(x):=frac{log(x)^4}{1+x}$$
          and pointwise $f_n rightarrow f$. Therefore, we can apply the monotone convergence theorem. However, the limes $f$ is not integrable, because
          $$int_{e}^infty f(x) , dx ge int_{e}^infty frac{1}{1+x} , dx =infty,$$
          and we cannot apply the dominated convergence theorem.







          share|cite|improve this answer












          share|cite|improve this answer



          share|cite|improve this answer










          answered Dec 4 '18 at 20:27









          p4schp4sch

          4,995217




          4,995217























              1












              $begingroup$

              To apply LDCT you need an $L^1$ majorant $g$ satisfying $|f_n(x)| le g(x)$ almost everywhere for all $n$. If $f_n to f$ almost everywhere then you have $|f(x)| le g(x)$ too.



              You can write $$f_n(x) = frac{n log^4 x}{n + nx + x^2} = frac{log^4 x}{1 + x + x^2/n}$$ so that $$f_n(x) to frac{log^4 x}{1+x}.$$
              This function is not integrable, so you won't find an integrable majorant $g$ with $|f_n| le g$ for all $n$. It looks like LDCT may not be the way to go.






              share|cite|improve this answer









              $endgroup$


















                1












                $begingroup$

                To apply LDCT you need an $L^1$ majorant $g$ satisfying $|f_n(x)| le g(x)$ almost everywhere for all $n$. If $f_n to f$ almost everywhere then you have $|f(x)| le g(x)$ too.



                You can write $$f_n(x) = frac{n log^4 x}{n + nx + x^2} = frac{log^4 x}{1 + x + x^2/n}$$ so that $$f_n(x) to frac{log^4 x}{1+x}.$$
                This function is not integrable, so you won't find an integrable majorant $g$ with $|f_n| le g$ for all $n$. It looks like LDCT may not be the way to go.






                share|cite|improve this answer









                $endgroup$
















                  1












                  1








                  1





                  $begingroup$

                  To apply LDCT you need an $L^1$ majorant $g$ satisfying $|f_n(x)| le g(x)$ almost everywhere for all $n$. If $f_n to f$ almost everywhere then you have $|f(x)| le g(x)$ too.



                  You can write $$f_n(x) = frac{n log^4 x}{n + nx + x^2} = frac{log^4 x}{1 + x + x^2/n}$$ so that $$f_n(x) to frac{log^4 x}{1+x}.$$
                  This function is not integrable, so you won't find an integrable majorant $g$ with $|f_n| le g$ for all $n$. It looks like LDCT may not be the way to go.






                  share|cite|improve this answer









                  $endgroup$



                  To apply LDCT you need an $L^1$ majorant $g$ satisfying $|f_n(x)| le g(x)$ almost everywhere for all $n$. If $f_n to f$ almost everywhere then you have $|f(x)| le g(x)$ too.



                  You can write $$f_n(x) = frac{n log^4 x}{n + nx + x^2} = frac{log^4 x}{1 + x + x^2/n}$$ so that $$f_n(x) to frac{log^4 x}{1+x}.$$
                  This function is not integrable, so you won't find an integrable majorant $g$ with $|f_n| le g$ for all $n$. It looks like LDCT may not be the way to go.







                  share|cite|improve this answer












                  share|cite|improve this answer



                  share|cite|improve this answer










                  answered Dec 4 '18 at 20:27









                  Umberto P.Umberto P.

                  38.8k13064




                  38.8k13064






























                      draft saved

                      draft discarded




















































                      Thanks for contributing an answer to Mathematics Stack Exchange!


                      • Please be sure to answer the question. Provide details and share your research!

                      But avoid



                      • Asking for help, clarification, or responding to other answers.

                      • Making statements based on opinion; back them up with references or personal experience.


                      Use MathJax to format equations. MathJax reference.


                      To learn more, see our tips on writing great answers.




                      draft saved


                      draft discarded














                      StackExchange.ready(
                      function () {
                      StackExchange.openid.initPostLogin('.new-post-login', 'https%3a%2f%2fmath.stackexchange.com%2fquestions%2f3026102%2ffind-lim-n-to-infty-int-mathbb-r-fracn-log4xnnxx2-chi-0%23new-answer', 'question_page');
                      }
                      );

                      Post as a guest















                      Required, but never shown





















































                      Required, but never shown














                      Required, but never shown












                      Required, but never shown







                      Required, but never shown

































                      Required, but never shown














                      Required, but never shown












                      Required, but never shown







                      Required, but never shown







                      Popular posts from this blog

                      How do I know what Microsoft account the skydrive app is syncing to?

                      When does type information flow backwards in C++?

                      Grease: Live!